You are on page 1of 32

Free Magazines & PDFs for

Quant Updater Banking Aspirants


02 May 2023 Maths by Arun Sir

702
MAR
MAY2023
2023| |SET
SET --59
27

1
mathsbyarunsir007@gmail.com WhatsApp @ 8445784192
maths by arun sir maths by arun sir maths by arun sir maths by arun sir
Free Magazines & PDFs for
Quant Updater Banking Aspirants
02 May 2023 Maths by Arun Sir

“जिसने संघर्ष को अपना साथी बना जिया


उसने ही इस दुजनया में
खुद को काबबि बना जिया.!!

मेरा नाम अरुण ससिं ह रावत हैं मैं साथियों जो BANK और Insurance Exams की परीक्षा देते है उन्हें Quant
ववषय समझाने में मदत करता हूँ
यूँहा आज मैं आपसे एक मेंटर की तरह नहीं बल्कि एक दोस्त की तरह बात कर रहा हूँ !
मेरी Journey भी आपकी ही तरह ही है की हर पररवार का सपना उनका बेटा या बेटी एक सरकारी नौकरी करे
तो मैथ्स के डर ने काफी लोग को इस रेस में हमेशा पीछे ही रखा है इस साल मैंने कोसशश की है हर बच्चे को बैंक
की तैयारी के थलए बैंक एग्जाम की फीस के बराबर ही कोसस फीस देनी पड़े ले वकन क्लास के बाद प्रैल्किस करने
के थलए content जब बच्चो को थमल नहीं पा रहा िा तो तो सोचा की इस पर भी काम करते है और सारे बच्चों के
थलए एक फ्री मैगज़ीन तैयार की है सजसका नाम है " QUANT UPDATER "
Dosto bahut acha lagta hai jab koi student message karta hai ki sir aapki guidance ki
wajah se mera exam qualify ho gya ya phir maine 35/35 Score kia hai. ye jaankar acha
lagta hai ki meri ki hui mehnat se kahin kisi ko help mil rahi hai aur uski success ka ek
chota sa hisaa mai bhi hu.

isi soch ko naya aayam dene ke liye mai lekar aaya hu " Quant Updater" jo ki ek magazine
hai jisme aapko daily sabhi quant topic jaise simplification, Approximation, number
series, data interpretation aur arithmetic ke questions practice karne ke liye milege. Iske
alwa aapko detailed solution dia gya hai jo aapki problems ko solve karega.

Dosto is magazine ki help se aap apni speed improve kar sakte ho aur apne quant ke dar
ko hara sakte ho. ab bhi koi result aaye aap apne sapne ko pura hota dekhe.

agar aapke koi doubt ya sawaal ho aap mujhe mail karke puch sakte ho

Dosto isi sapne ke saath ki is saal aap sabhi selection lekar message karoge…

Dosti is magazine ki help se mai chahta hu ki aapko jyada se jyada practice kara paun
taki aap bhi apne parents ko bol paye han maine kar dikhaya………….

आपका दोस्त | आपका मेंटर


अरुण ससिं ह रावत
( Maths by Arun Sir )
8445784192
Mathsbyarunsir007@gmail.com

2
mathsbyarunsir007@gmail.com WhatsApp @ 8445784192
maths by arun sir maths by arun sir maths by arun sir maths by arun sir
Free Magazines & PDFs for
Quant Updater Banking Aspirants
02 May 2023 Maths by Arun Sir

3
mathsbyarunsir007@gmail.com WhatsApp @ 8445784192
maths by arun sir maths by arun sir maths by arun sir maths by arun sir
Free Magazines & PDFs for
Quant Updater Banking Aspirants
02 May 2023 Maths by Arun Sir

CONTENT
INDEX
Simplification & Approximation

1. Question Paper …………………………….. 5-6

2. Solution ……………………. 6-7

Number Series | Missing & Wrong Series

1. Question Paper …………………………….. 7-8

2. Solution ……………………. 8-9

Quadratic Equation

1. Question Paper …………………………….. 9

2. Solution ……………………. 9-12

Data Interpretation

1. Question Paper …………………………….. 12-17

2. Solution ……………………. 17- 18

Arithmetic

1. Question Paper …………………………….. 18-20

2. Solution ……………………. 21-23

4
mathsbyarunsir007@gmail.com WhatsApp @ 8445784192
maths by arun sir maths by arun sir maths by arun sir maths by arun sir
Free Magazines & PDFs for
Quant Updater Banking Aspirants
02 May 2023 Maths by Arun Sir

Chapter - 1 | Simplifications & Approximations Based Questions


1.What will come in the place of question mark (?) in the given expression?
1. ?2/3 = 8 × 28 – 55 × 4
A8
B4
C 12
D 16
E None of these
2What will come in the place of question mark (?) in the given expression?
156% of 198% of ? = 312% of 396% of 128
A 572
B 512
C 536
D 548
E 500
3. What will come in the place of question mark (?) in the given expression?
(47 × 8 – 17 × ?) = 189
A 19
B 12
C 17
D 11
E None of these
4. What will come in the place of question mark (?) in the given expression?
? = 25% of 75% of 4000 + 50 × 3
A 965
B 945
C 925
D 900
E 875
5. What will come in the place of question mark (?) in the given expression?
?2 = 1080 + 189 × 5
A 45
B 55
C 35
D 65
E 25
6.What approximate value will come in place of the question mark (?) in the following question?(Note: You are
not expected to calculate the exact value.)
(9.98% of 599.98) ÷ 14.99 = ? – 11.98% of 299.99

5
mathsbyarunsir007@gmail.com WhatsApp @ 8445784192
maths by arun sir maths by arun sir maths by arun sir maths by arun sir
Free Magazines & PDFs for
Quant Updater Banking Aspirants
02 May 2023 Maths by Arun Sir

A -14
B 96
C 17
D 73
E 40
7.What approximate value will come in place of the question mark (?) in the following question?(Note: You are
not expected to calculate the exact value.)
19.98% of 819.99 = 24.98% of ?
A 580
B 656
C 748
D 832
E 964
8. What approximate value will come in place of the question mark (?) in the following question?(Note: You are
not expected to calculate the exact value.)
? = (40.16% of 24.95% of 1479.58) + 254.33
A 268
B 312
C 354
D 402
E 584
9.What approximate value will come in place of the question mark (?) in the following question?(Note: You are
not expected to calculate the exact value.)
? = 28.03 × 44.81 ÷ 5.03 + (7.93)2.001
A 316
B 228
C 94
D 438
E 189
10. What approximate value will come in place of the question mark (?) in the following question?(Note: You are
not expected to calculate the exact value.)
? = (23.12 + 36.88)% of 33.33% of 2439.78
A 356
B 618
C 556
D 412
E 488

6
mathsbyarunsir007@gmail.com WhatsApp @ 8445784192
maths by arun sir maths by arun sir maths by arun sir maths by arun sir
Free Magazines & PDFs for
Quant Updater Banking Aspirants
02 May 2023 Maths by Arun Sir

1Solution 60 ÷ 15 ~ ? – 36
?2/3 = 8 × 28 – 55 × 4 ? ~ 4 + 36
?2/3
= 224 – 220 ? ~ 40
?2/3
=4 Hence, option e.
?2 = 64 Previous
?=8 7Solution
Hence, option a. 19.98% of 819.99 = 24.98% of ?
Previous 20% of 820 ~ 25% of ?
2Solution 0.25 × ? ~ 0.20 × 820
156% of 198% of ? = 312% of 396% of 128 0.25 × ? ~ 164
? = 2 × 2 × 128 ? ~ 656
? = 128 × 4 = 512 Hence, option b.
Hence, option b. Previous
Previous 8Solution
3Solution ? = (40.16% of 24.95% of 1479.58) + 254.33
(47 × 8 – 17 × ?) = 189 ? ~ (40% of 25% of 1480) + 254
17 × ? = 187 ? ~ 0.1 × 1480 + 254
? = 11 ? ~ 148 + 254
Hence, option d. ? ~ 402
Previous Hence, option d.
4Solution Previous
? = 25% of 75% of 4000 + 50 × 3 9Solution
? = 0.25 × 0.75 × 4000 + 50 × 3 ? = 28.03 × 44.81 ÷ 5.03 + (7.93)2.001
? = 750 + 150 ? ~ (28 × 45) ÷ 5 + 82
? = 900 ? ~ 252 + 64
Hence, option d. ? ~ 316
Previous Hence, option a.
5Solution Previous
? = 1080 + 189 × 5
2
10Solution
? = 1080 + 945
2
? = (23.12 + 36.88)% of 33.33% of 2439.78
?2 = 2025 ? ~ (23 + 37)% of 1/3 × 2440
? = 45 ? ~ 3/5 × 1/3 × 2440
Hence, option a. ? ~ 488
Previous Hence, option e.
6Solution Previous
(9.98% of 599.98) ÷ 14.99 = ? – 11.98% of 299.99
(10% of 600) ÷ 15 ~ ? – 12% of 300
(0.10 × 600) ÷ 15 ~ ? – 0.12 × 300

7
mathsbyarunsir007@gmail.com WhatsApp @ 8445784192
maths by arun sir maths by arun sir maths by arun sir maths by arun sir
Free Magazines & PDFs for
Quant Updater Banking Aspirants
02 May 2023 Maths by Arun Sir

Chapter - 2 | Missing & Wrong Number Series Based Questions


11. What will come in place of the question mark (?) in the following series?
5, 6, 13, 40, ?, 806
A 127
B 213
C 188
D 161
E 240
12.What will come in place of the question mark (?) in the following series?
288, 144, 72, ?, 18, 9
A 36
B 48
C 40
D 24
E 56
13. What will come in place of the question mark (?) in the following series?
12, 16, 25, 41, 66, ?
A 116
B 108
C 124
D 112
E 102
14. What will come in place of the question mark (?) in the following series?
63, 68, 78, ?, 113, 138
A 89
B 93
C 97
D 85
E 90
15. What will come in place of the question mark (?) in the following series?
47, 26, 68, ?, 89, -16
A 26
B 15
C5
D 18
E None of these
16. The following numbers form a series. Find the odd one out.

8
mathsbyarunsir007@gmail.com WhatsApp @ 8445784192
maths by arun sir maths by arun sir maths by arun sir maths by arun sir
Free Magazines & PDFs for
Quant Updater Banking Aspirants
02 May 2023 Maths by Arun Sir

12, 22, 43, 75, 108, 172


A 22
B 43
C 75
D 108
E 172
17.The following numbers form a series. Find the odd one out.
227, 222, 210, 199, 157, 112
A 222
B 210
C 199
D 157
E 112
18. The following numbers form a series. Find the odd one out.
7, 5, 12, 34, 138, 692
A5
B 12
C 34
D 138
E 692
19. The following numbers form a series. Find the odd one out.
9, 13, 22, 47, 98, 217
A 13
B 22
C 47
D 98
E 217
20. The following numbers form a series. Find the odd one out.
7, 15, 31, 63, 87, 127
A 15
B 31
C 63
D 87
E 127

9
mathsbyarunsir007@gmail.com WhatsApp @ 8445784192
maths by arun sir maths by arun sir maths by arun sir maths by arun sir
Free Magazines & PDFs for
Quant Updater Banking Aspirants
02 May 2023 Maths by Arun Sir

11Solution Hence, option c.


5×1+1=6 Previous
6 × 2 + 1 = 13 16Solution
13 × 3 + 1 = 40 12 + 10 = 22
40 × 4 + 1 = 161 22 + 21 = 43
161 × 5 + 1 = 806 43 + 32 = 75
Hence, option d. 75 + 43 = 118
Previous 118 + 54 = 172
12Solution Therefore, 118 should be in place of 108
288 ÷ 2 = 144 Hence, option d.
144 ÷ 2 = 72 Previous
72 ÷ 2 = 36 17Solution
36 ÷ 2 = 18 227 – 5 × 1 = 222
18 ÷ 2 = 9 222 – 6 × 2 = 210
Hence, option a. 210 – 7 × 3 = 189
Previous 189 – 8 × 4 = 157
13Solution 157 – 9 × 5 = 112
12 + 22 = 16 Therefore, 189 should be in place of 199
16 + 3 = 25
2
Hence, option c.
25 + 4 = 41
2
Previous
41 + 52 = 66 Solution
66 + 6 = 102
2
7×1–2=5
Hence, option e. 5 × 2 + 2 = 12
Previous 12 × 3 – 2 = 34
14Solution 34 × 4 + 2 = 138
63 + 5 = 68 138 × 5 – 2 = 688
68 + 10 = 78 Therefore, 692 should be in place of 688.
78 + 15 = 93 Hence, option e.
93 + 20 = 113 Previous
113 + 25 = 138 19.Solution
Hence, option b. 9 + 22 = 13
Previous 13 + 32 = 22
15.Solution 22 + 52 = 47
47 – 21 = 26 47 + 72 = 96
26 + 42 = 68 96 + 112 = 217
68 – 63 = 5 Therefore, 96 should be in place of 98.
5 + 84 = 89 Hence, option d.
89 – 105 = -16 Preious

10
mathsbyarunsir007@gmail.com WhatsApp @ 8445784192
maths by arun sir maths by arun sir maths by arun sir maths by arun sir
Free Magazines & PDFs for
Quant Updater Banking Aspirants
02 May 2023 Maths by Arun Sir

20Solution 87 + 4 × 10 = 127
7 + 4 × 2 = 15 Therefore, 55 should be in place of 63.
15 + 4 × 4 = 31 Hence, option c.
31 + 4 × 6 = 55 Previous
55 + 4 × 8 = 87

Chapter - 3 | Quadratic Equations Based Questions


21-30. In the question, two equations I and II are given. You have to solve both the equations to establish the
correct relation between x and y and choose the correct option.
A. x > y B. x < y C. x = y or the relationship cannot be established
D. x ≥ y E. x ≤ y
2I. I. x2 + 21x + 108 = 0 II. y2 + 17y + 72 = 0
22. I. x2 + 11x + 28 = 0 II. y2 + 15y + 56 = 0
23. I. x2 – 20x + 96 = 0 II. y2 – 21y + 90 = 0
24. I. x2 – 31x + 240 = 0 II. y2 + 3y – 270 = 0
25. I. x2 + 11x + 28 = 0 II. y2 + 20y + 96 = 0
26. I. 12x2 – 5x – 23 = -16 II. 12y2 = 7y
27. I. x2 – 6x – 112 = 0 II. y2 + 22y + 120 = 0
28. I. 10x2 – 47x + 42 = 0 II. 5y2 – y – 6 = 0
29. I. x2 - 5x - 84 = 0 II. y2 - 31y + 234 = 0
30. I. x2 – 19x + 84 = 0 II. y2 – 18y + 72 = 0

21. Solution
-12 < -8
From I:
x2 + 21x + 108 = 0 -12 < -9
x + 9x + 12x + 108 = 0
2

So, x ≤ y
x(x + 9) + 12(x + 9) = 0
Hence, option e.
(x + 9)(x + 12) = 0
Previous
x = -9, -12
22. Solution
From II:
From I:
y2 + 17y + 72 = 0
x2 + 11x + 28 = 0
y2 + 8y + 9y + 72 = 0
x2 + 7x + 4x + 28 = 0
y(y + 8) + 9(y + 8) = 0
x(x + 7) + 4(x + 7) = 0
(y + 8)(y + 9) = 0
(x + 7)(x + 4) = 0
y = -8, -9
x = -7, -4
X Relation y From II:
y2 + 15y + 56 = 0
-9 < -8
y2 + 8y + 7y + 56 = 0
y(y + 8) + 7(y + 8) = 0
-9 = -9
(y + 8)(y + 7) = 0

11
mathsbyarunsir007@gmail.com WhatsApp @ 8445784192
maths by arun sir maths by arun sir maths by arun sir maths by arun sir
Free Magazines & PDFs for
Quant Updater Banking Aspirants
02 May 2023 Maths by Arun Sir

y = -7, -8 x2 – 31x + 240 = 0


x2 – 15x – 16x + 240 = 0
X Relation Y
x(x – 15) – 16(x – 15) = 0
-7 > -8 (x – 15)(x – 16) = 0
x = 15, 16
-7 = -7 From II:
y2 + 3y – 270 = 0
-4 > -8
y2 + 18y – 15y – 270 = 0
y(y + 18) – 15(y + 18) = 0
-4 > -7
(y + 18)(y – 15) = 0
So, x ≥ y y = -18, 15
Hence, option d.
X Relation Y
Previous
23Solution
15 > -18
From I:
x2 – 20x + 96 = 0 16 > 15
x2 – 8x – 12x + 96 = 0
x(x – 8) – 12(x – 8) = 0 15 = 15
(x – 8)(x – 12) = 0
16 > -18
x = 8, 12
From II: So, x ≥ y
y – 21y + 90 = 0
2
Hence, option d.
y2 – 6y – 15y + 90 = 0 Previous
y(y – 6) – 15(y – 6) = 0 25.Solution
(y – 6)(y – 15) = 0 From I:
y = 6, 15 x2 + 11x + 28 = 0
x2 + 7x + 4x + 28 = 0
X Relation Y
x(x + 7) + 4(x + 7) = 0
8 > 6 (x + 7)(x + 4) = 0
x = -7, -4
8 < 15 From II:
y2 + 20y + 96 = 0
12 > 6
y2 + 8y + 12y + 96 = 0
y(y + 8) + 12(y + 8) = 0
12 < 15
(y + 8)(y + 12) = 0
So, relationship between x and y cannot be established y = -8, -12
Hence, option c.
X Relation y
Previous
24. Solution
-7 > -8
From I:

12
mathsbyarunsir007@gmail.com WhatsApp @ 8445784192
maths by arun sir maths by arun sir maths by arun sir maths by arun sir
Free Magazines & PDFs for
Quant Updater Banking Aspirants
02 May 2023 Maths by Arun Sir

y2 + 22y + 120 = 0
-7 > -12
y2 + 10y + 12y + 120 = 0
-4 > -8 y(y + 10) + 12(y + 10) = 0
y = -10, -12
-4 > -12
X Relation Y
So, x > y
Hence, option a. 14 > -10
Previous
14 > -12
26. Solution
From I:
-8 > -10
12x2 – 5x – 23 = -16
12x2 – 5x – 7 = 0 -8 > -12
12x2 – 12x + 7x – 7 = 0
So, x > y
12x(x – 1) + 7(x – 1) = 0
Hence, option a.
(12x + 7)(x – 1) = 0
Previous
x = (-7/12), 1
28.Solution
From II:
From I:
12y2 = 7y
10x2 – 47x + 42 = 0
y = 7/12, 0
10x2 – 12x – 35x + 42 = 0
X Relation Y 2x(5x – 6) – 7(5x – 6) = 0
(2x – 7)(5x – 6) = 0
-7/12 < 7/12
x = 7/2, 6/5
x = 3.5, 1.2
-7/12 < 0
From II:
1 > 7/12 5y2 – y – 6 = 0
5y2 – 6y + 5y – 6 = 0
1 > 0 y(5y – 6) + 1(5y – 6) = 0

So, no relation can be established between x and y. (5y – 6)(y + 1) = 0

Hence, option c. y = 6/5, -1

Previous y = 1.2, -1

27. Solution X Relation Y


From I:
x2 – 6x – 112 = 0 3.5 > 1.2
x2 – 14x + 8x – 112 = 0
3.5 > -1
x(x – 14) + 8(x – 14) = 0
(x – 14)(x + 8) = 0
1.2 = 1.2
x = 14, -8
From II: 1.2 > -1

13
mathsbyarunsir007@gmail.com WhatsApp @ 8445784192
maths by arun sir maths by arun sir maths by arun sir maths by arun sir
Free Magazines & PDFs for
Quant Updater Banking Aspirants
02 May 2023 Maths by Arun Sir

So, x ≥ y x2 – 19x + 84 = 0
Hence, option d. x2 – 7x – 12x + 84 = 0
Previous x(x – 7) – 12(x – 7) = 0
29. Solution (x – 12)(x – 7) = 0
I.x2 - 5x - 84 = 0 x = 12, 7
=> x2 - 12x + 7x - 84 = 0 From II:
=> (x - 12)(x + 7) = 0
=> x = 12 or -7 y2 – 18y + 72 = 0
II. y2 - 31y + 234 = 0 y2 – 6y – 12y + 72 = 0
=> y2 - 13y - 18y + 234 = 0 y(y – 6) – 12(y – 6) = 0
=> (y - 13)(y - 18) = 0
(y – 6)(y – 12) = 0
=> y = 13 or 18
When x = 12 and y = 13, so x < y y = 6, 12
When x = 12 and y = 18, so x < y
When x = -7 and y = 13, so x < y X Relation Y
When x = -7 and y = 18, so x < y
Hence, x < y 12 > 6

Previous
12 = 12
30. Solution
From I:

Chapter - 4 | Data Interpretation


Q31-36. Study the following information carefully and answer the questions given beside.
Information about number of patients who were tested positive to COVID-19 tests in five different locations
of India is as follows.
Mumbai has 60% more patients than Jodhpur, which has 400 more than China. Number of patients in
Calcutta was half the number of patients in China. Number of patients in Gujrat was 100 less than China.
Total patients were 9100 as on 30 april 2019 in all the five cities together.
It was found that out of every 200 patients, 180 recovered within 14 days, 18 took 30 days to recover and 2
died.
ददशाथनदेश: दी गई जानकारी का ध्यानपूवसक अध्ययन करेूँ और ददए गए प्रश्नों के उत्तर दें।
भारत के पाांच अलग-अलग शहरों में COVID-19 परीक्षणों में सकारात्मक पाए गए रोगगयों की सांख्या के बारे में जानकारी
थनम्नानुसार है।
जोधपुर की तुलना में मुांबई में 60% अगधक मरीज हैं , और जोधपुर में चीन से 400 अगधक मरीज हैं। कलकत्ता में रोगगयों की सांख्या
चीन में रोगगयों की सांख्या से आधी िी। गुजरात में मरीजों की सांख्या चीन से 100 कम िी। 30 अप्रैल 2019 तक पाांचो शहरों में कुल
मरीज की सांख्या 9100 िी।
यह पाया गया है वक प्रत्येक 200 रोगगयों में से 14 ददनों के भीतर 180 ठीक हुए, 18 को ठीक होने में 30 ददन लगे और 2 की मृत्यु हो
गई।
Q31. Find average number of patients in China, Calcutta and Gujrat.
चीन, कलकत्ता और गुजरात में रोगगयों की औसत सांख्या ज्ञात कीसजए।
A. 1100
B. 1200

14
mathsbyarunsir007@gmail.com WhatsApp @ 8445784192
maths by arun sir maths by arun sir maths by arun sir maths by arun sir
Free Magazines & PDFs for
Quant Updater Banking Aspirants
02 May 2023 Maths by Arun Sir

C. 1300
D. 1400
E. None of these
Q32. Number of patients in Jodhpur was what percent more than Calcutta?
जोधपुर में रोगगयों की सांख्या कलकत्ता से वकतने प्रवतशत अगधक िी?
A. 100%
B. 150%
C. 200%
D. 250%
E. None of these
Q33. Find average number of patients in Calcutta and Gujrat.
कलकत्ता और गुजरात में रोगगयों की औसत सांख्या ज्ञात कीसजए।
A. 1100
B. 1200
C. 1150
D. 1400
E. None of these
Q34. For each 1000 tests the numbers of people who were found positive were 130. Find out how many tests were
conducted that produced 9100 total positive cases?
प्रत्येक 1000 परीक्षणों करने पर पॉसजरटव पाए गए लोगों की सांख्या 130 िी। पता करें वक 9100 पॉसजरटव मामलों को पता करने के
थलए कुल वकतने परीक्षण वकए गए िे?
A. 35000
B. 40000
C. 91000
D. 130000
E. 70000
Q35. How many patients recovered till 31 may 2020, if all the patients in Mumbai, Jodhpur and Calcutta are
considered?
मुांबई, जोधपुर और कलकत्ता के सभी रोगगयों को जोड़ा जाए तो 31 मई 2020 तक वकतने मरीज ठीक हुए?
A. 5400
B. 5540
C. 4590
D. 5940
E. 5990
Q36. How many people died in Jodhpur, Mumbai and China together?
जयपुर, मुांबई और चीन में एक साि वकतने लोगों की मृत्यु हुई?
A. 41
B. 51

15
mathsbyarunsir007@gmail.com WhatsApp @ 8445784192
maths by arun sir maths by arun sir maths by arun sir maths by arun sir
Free Magazines & PDFs for
Quant Updater Banking Aspirants
02 May 2023 Maths by Arun Sir

C. 55
D. 112
E. 102
Directions (37-41): Answer the questions based on the information given below.
The given table shows the difference between number of Mobiles (M) sold and number of Chargers (C) sold,
and ratio of number of Mobiles(M) sold to number of Chargers (C) sold, by a company in given 5 Days.
Note: In each month, total number of items sold = number of mobiles sold + number of chargers sold
जनदेश: नीचे दी गई जानकारी के आधार पर प्रश्नों के उत्तर दें।
दी गई ताथलका में एक कांपनी द्वारा 5 ददनों में बेचे गए mobiles (M) और बेचे गए chargers (C) की सांख्या के बीच का अांतर और बेचे
गए mobiles (M) की सांख्या और बेचे गए chargers (C) की सांख्या के अनुपात को दशासया गया है।
नोट: प्रत्येक ददन में बेचे गए सामान की कुल सांख्या = बेचे गए mobiles की सांख्या + बेचे गए chargers की सांख्या

Difference between number of Ratio of number of


Mobiles(M) sold and number Mobiles(M) sold to number of
Days Chargers (C) sold Chargers (C) sold

Monday 190 9:7

Tuesday 240 7:5

Wednesday 360 9:5

Thursday 180 13:10

Friday 110 8:7

37. What is the ratio between sum of number of M sold in the Tuesday and Wednesday to the sum of number C
sold in the Wednesday and Thursday ?
मांगलवार और बुधवार में बेचे गए M की सांख्या के योग तिा बुधवार और गुरुवार में बेची गई C की सांख्या के योग के बीच का अनुपात
वकतना है?
A 12:17
B 8:13
C 7:11
D 11:7
E 17:12
38. What is the average of the total number of items sold in Tuesday and Friday ?

मांगलवार और शुक्रवार में बेचे गए सामान की कुल सांख्या का औसत क्या है?

A 1480

16
mathsbyarunsir007@gmail.com WhatsApp @ 8445784192
maths by arun sir maths by arun sir maths by arun sir maths by arun sir
Free Magazines & PDFs for
Quant Updater Banking Aspirants
02 May 2023 Maths by Arun Sir

B 1545

C 1550

D 1580

E 1495

39. Total number of M sold in the day of Thursday and Friday , together is how many more than the total number
of items sold in the day of Monday?

गुरुवार और शुक्रवार के ददन में बेचे गए M की कुल सांख्या, सोमवार के ददन में बेचे गए सामान की कुल सांख्या से वकतनी अगधक है?

A 220

B 250

C 140

D 110

E 190

40. Number of M sold in the day of Tuesday is how much percent less than the total number of C sold in the day
of Tuesday and Thursday ?

मांगलवार के ददन में बेचे गए M की सांख्या मांगलवार और गुरुवार के ददन में बेची गई C की कुल सांख्या से वकतने प्रवतशत कम है?

A 25%

B 28%

C 40%

D 32%

E 30%

41. The difference between the number of M and C sold in another day Saturday, is 62.5% less than that in the day
of Wednesday . If the ratio of number of M sold to number of C sold, in the day of Saturday is 14:11, then find the
total number of items (M + C) sold in the day of Saturday?

एक अन्य ददन शथनवार में बेचे गए M और C की सांख्या के बीच का अांतर बुधवार के ददन की तुलना में 62.5% कम है। यदद शथनवार के ददन
में बेचे गए M की सांख्या और बेचे गए C की सांख्या का अनुपात 14:11 है, तो शथनवार के ददन में बेचे गए सामान (M + C) की कुल सांख्या
ज्ञात कीसजए?

A 1125

B 1245

C 1315

D 1075

E 945

Directions(42-46): Answer the questions based on the information given below.


The given line graph shows the number of pencil sold and percentage of number of pencil sold out of total
number of writing material (pencil + pen) sold by given 5 shops namely A,B,C,D and E.
जनदेश: नीचे दी गई जानकारी के आधार पर प्रश्नों के उत्तर दें।

17
mathsbyarunsir007@gmail.com WhatsApp @ 8445784192
maths by arun sir maths by arun sir maths by arun sir maths by arun sir
Free Magazines & PDFs for
Quant Updater Banking Aspirants
02 May 2023 Maths by Arun Sir

ददए गए लाइन ग्राफ में 5 दुकानों A,B,C,Dऔर E द्वारा बेची गई पेंससल की सांख्या और बेचे गए ले खन सामग्री (पेंससल + पेन ) की कुल
सांख्या में बेची गई पेंससल की सांख्या का प्रवतशत दशासया गया है।

number of pencil sold

percentage of number of pencil sold out of total number of writing material sold

1800

1600
1560
1400
1260
1200

1000 1056

800
700
600 640

400

200

0 62.50% 75% 52% 55% 50%


A B Category 4 D E

42. The sum of number of pencil sold by A and B is how many more than number of pen sold by D?
A और B द्वारा बेची गई पेंससल की सांख्या का योग D द्वारा बेचे गए पेन की सांख्या से वकतना अगधक है?
A 912
B 654
C 1096
D 1232
E 844
43. What is the ratio between total number of pen sold by shop B and E together and total number of writing
material sold by shop C?
दुकान B और E द्वारा बेचे गए पेन की कुल सांख्या और दुकान C द्वारा बेचे गए ले खन सामग्री की कुल सांख्या के बीच का अनुपात वकतना
है?
A 52:179
B 53:150
C 51:160
D 50:131
E None of these
44. Number of pencil sold by shop D is how much percent more than the number of pen sold by shop D?
दुकान D द्वारा बेची गई पेंससल की सांख्या दुकान D द्वारा बेचे गए पेन की सांख्या से वकतने प्रवतशत अगधक है?
A (250/7)%
B (175/4)%

18
mathsbyarunsir007@gmail.com WhatsApp @ 8445784192
maths by arun sir maths by arun sir maths by arun sir maths by arun sir
Free Magazines & PDFs for
Quant Updater Banking Aspirants
02 May 2023 Maths by Arun Sir

C (87/5)%
D (133/6)%
E (200/9)%
45. What is the average of the total number of writing material sold by shops A and D?
दुकान A और D द्वारा बेचे गए ले खन सामग्री की कुल सांख्या का औसत वकतना है?
A 1440
B 1300
C 1280
D 1520
E 1640
46. Number of pen sold by another shop G is 25% more than thaE by shop C. Find the difference between number
pen sold by shop A and shop G.
एक अन्य दुकान G द्वारा बेचे गए पेन की सांख्या दुकान C द्वारा बेचे गए पेन की सांख्या से 25% अगधक है। दुकान A और दुकान G द्वारा बेचे
गए पेन की सांख्या के बीच का अांतर ज्ञात कीसजए।
A 1420
B 1560
C 1160
D 1240
E 1380
ARITHMETIC QUESTIONS :-
Q47. Two pipes can fill a tank in 18 minutes and 27 minutes. A third pipe can empty the tank when it is full in 6
minutes. All the three pipes are opened when the tank was 2/3rd full. In how many minutes will the tank
become empty?
दो पाइप क्रमशः 18 जमनट और 27 जमनट में एक टं की को भर सकते हैं । तीसरी पाइप 6 जमनट में भरी हुई टं की को खािी कर सकती
है । टं की का 2/3 हहस्सा भरे होने पर, सभी तीन पाइपों को खोिा िाता है । टं की बकतने जमनट में खािी हो िाएगी?
A) 9
B) 18
C) 15
D) 14
E) 20
Q48 Radhika got 43 marks in Hindi, 45 marks in Science, 67 marks in Maths, 89 marks in Social Science and 65
marks in English. The maximum marks of each subject are 120. How much overall percentage of marks did
she get?
Radhika को रहन्दी में 43, ववज्ञान में 45, गसणत में 67, समाज ववज्ञान में 89 और अांग्रेजी में 65 अांक थमले हैं। प्रत्येक ववषय का अगध
कतम अांक 120 हैं। उसे कुल वकतने प्रवतशत अांक थमले हैं ?
A) 55.1
B) 51.5
C) 65
D) 62

19
mathsbyarunsir007@gmail.com WhatsApp @ 8445784192
maths by arun sir maths by arun sir maths by arun sir maths by arun sir
Free Magazines & PDFs for
Quant Updater Banking Aspirants
02 May 2023 Maths by Arun Sir

E) None of these
Q49. Sagar can row 12 km in 7 hrs. in downstream and 8 km in 5 hrs. in upstream. Find the time required to cover
116 km in still water by him.
Sagar नाव के द्वारा धारा की ददशा में 12 वकमी की दूरी 7 घांटे में तय करता है और धारा की ववपरीत ददशा में 8 वकमी की दूरी 5 घांटे
में तय करता है। उसके द्वारा स्थिर जल में 116 वकमी की दूरी तय करने में वकतना समय लगेगा?
70 hrs 64 hrs 72 hrs 58 hrs None of these
Q50. Ramesh is 8 years older than Mukesh while Mukesh is 6 years older than Sandeep. If ratio of age of Sandeep
to age of Ramesh is 5: 12, respectively, then what will be the age of Mukesh after 12 years?
Ramesh Mukesh से 8 वषस बड़ा है जबवक Mukesh Sandeep से 6 वषस बड़ा है। यदद Sandeep की आयु का Ramesh की आयु
से अनुपात क्रमशः 5:12 है, तो 12 वषस के बाद Mukesh की आयु वकतनी होगी?
26 years 24 years 28 years 27 years 30 years
Q51. Half of Sanjeev’s monthly income is equal to four-seventh of Rahul’s monthly income. Rajni’s monthly
income is Rs 64000 which is double the monthly income of Sanjeev. What is Rahul’s monthly income?
Sanjeev की माससक आय का आधा Rahul की माससक आय के 4/7 के बराबर है। Rajni की माससक आय 64000 रुपये जो
Sanjeev की माससक आय का डबल है तो Rahul की माससक आय क्या है?
A) Rs. 36000
B) Rs. 24570
C) Rs. 28000
D) Cannot be determined
E) None of these
52. Vinit and Punit started a business by investing their capitals in the ratio of 7:9, respectively. After 8 months,
Vinit decreased his investment by Rs. 500 and Punit increased his investment by Rs. 500. Find the initial investment
made by Punit if the annual profit share of Vinit is Rs. 1400 out of total annual profit of Rs. 3360.
ववनीत और पुनीत ने क्रमशः 7:9 के अनुपात में अपनी पूांजी थनवेश करके एक व्यवसाय शुरू वकया। 8 महीने के बाद, ववनीत ने अपने थनवेश
में से Rs. 500 की कमी की और पुनीत ने अपने थनवेश में Rs. 500 की वृद्धि की। पुनीत द्वारा वकए गए प्रारांथभक थनवेश ज्ञात कीसजए, यदद
कुल वावषि क लाभ Rs. 3360 में से ववनीत का वावषि क लाभ Rs. 1400 है।
A Rs. 7200
B Rs. 4500
C Rs. 8100
D Rs. 3600
E Rs. 10800
53. Jar ‘A’ (milk + water) contains milk and water in the ratio 17:11, respectively. 60% of this mixture has been taken
out and poured in another jar ‘B’ which already contains 21 litres of milk and 75 litres of water. Find the initial
quantity of mixture in jar ‘A’, if the final ratio of quantities of milk and water, in jar ‘B’ is 2:3, respectively.
जार 'A' (दूध + पानी) में दूध और पानी का अनुपात क्रमश: 17:11 है। इस थमश्रण के 60% भाग को थनकाल कर दूसरे जार 'B' में डाल ददया
गया है सजसमें पहले से ही 21 liters दूध और 75 liters पानी है। जार 'A' में थमश्रण की प्रारांथभक मात्रा ज्ञात कीसजए, यदद जार 'B' में दूध और
पानी की मात्रा का अांवतम अनुपात क्रमशः 2:3 है।
A 196 litres
B 280 litres

20
mathsbyarunsir007@gmail.com WhatsApp @ 8445784192
maths by arun sir maths by arun sir maths by arun sir maths by arun sir
Free Magazines & PDFs for
Quant Updater Banking Aspirants
02 May 2023 Maths by Arun Sir

C 224 litres
D 140 litres
E 112 litres
54. A work can be completed by ‘P’ alone and ‘Q’ alone in 32 days and 28 days, respectively. ‘P’ started the work
alone and worked for 2 consecutive days and then on third day, ‘Q’ replaced ‘P’, and then for next two days, ‘P’
again replaced ‘Q’ and the process goes on. Find the number of days taken to complete the work this way.
एक कायस को अकेले 'P' और अकेले 'Q' द्वारा क्रमशः 32 ददनों और 28 ददनों में पूरा वकया जा सकता है। 'P' अकेले कायस शुरू करता है और
लगातार 2 ददनों तक कायस करता है और दफर तीसरे ददन, 'Q' से 'P' को बदला जाता है और दफर अगले दो ददनों के थलए, 'P' से 'Q' को बदला
जाता है और यही प्रवक्रया ऐसे ही आगे भी वकया जाता है। इस प्रकार कायस को पूरा करने में लगने वाले ददनों की सांख्या ज्ञात कीसजए।
A (214/7) days
B (200/9) days
C (185/12) days
D (162/7) days
E (238/9) days
55. Present average age of Tina and Rina is 30 years. 2 years hence from now, age of Rina and Mina will be in the
ratio of 9:8, respectively. What is the present age of Rina, if the ratio of age of Tina 2 years hence from now and
age of Mina 9 years ago from now is 4:3, respectively.
टीना और रीना की वतसमान औसत आयु 30 वषस है। अब से 2 वषस बाद, रीना और मीना की आयु क्रमशः 9:8 के अनुपात में होगी। रीना की
वतसमान आयु क्या है, यदद अब से 2 वषस बाद टीना की आयु और अब से 9 वषस पहले मीना की आयु का अनुपात क्रमशः 4:3 है।
A 38 years
B 34 years
C 24 years
D 42 years
E 48 years
56. An article having cost price of Rs. 2400 is marked 50% above its cost price. Find the selling price of the article
if the discount offered on the article is Rs. 240 more than the profit earned on it.
एक वस्तु, सजसका क्रय मूल्य Rs. 2400 है, को इसके क्रय मूल्य से 50% अगधक अांवकत वकया गया है। वस्तु का ववक्रय मूल्य ज्ञात कीसजए
यदद वस्तु पर दी जाने वाली छूट उस पर असजि त लाभ से Rs. 240 अगधक है।
A Rs. 3060
B Rs. 2440
C Rs. 2050
D Rs. 2880
E Rs. 3120
57. Total surface area and curved surface area of a cone is 600 cm2 and 408 cm2. Find the volume of a cylinder
whose height is 15 cm and radius is 75% more than that of the cone. (Take π = 3)
एक शांकु का कुल पृष्ठीय क्षेत्रफल और वक्र पृष्ठीय क्षेत्रफल 600 cm2 और 408 cm2 है। एक बेलन का आयतन ज्ञात कीसजए सजसकी
ऊूँचाई 15 cm और वत्रज्या शांकु की वत्रज्या से 75% अगधक है। (π = 3 )
A 9415 cm3
B 6884 cm3

21
mathsbyarunsir007@gmail.com WhatsApp @ 8445784192
maths by arun sir maths by arun sir maths by arun sir maths by arun sir
Free Magazines & PDFs for
Quant Updater Banking Aspirants
02 May 2023 Maths by Arun Sir

C 5432 cm3
D 7536 cm3
E 8820 cm3
58. Train ‘A’ can cross train ‘B’ in 8 seconds while travelling in opposite direction whereas it takes 32 seconds to
cross train ‘B’ while travelling in same direction. Find the distance travelled by train ‘A’ in 5 hours if the speed of
train ‘B’ is 54 km/hr.
ट्रे न 'A' ववपरीत ददशा में यात्रा करते हुए ट्रे न 'B' को 8 seconds में पार कर सकती है जबवक समान ददशा में यात्रा करते हुए ट्रे न 'B' को पार
करने में 32 seconds का समय ले ती है। यदद ट्रे न 'B' की गवत 54 km/hr है, तो ट्रे न 'A' द्वारा 5 hours में तय की गई दूरी ज्ञात कीसजए।
A 720 km
B 540 km
C 450 km
D 360 km
E 270 km
59. Monthly incomes of Axar and Avesh are in the ratio 7:10, respectively while the ratio of their monthly
expenditures is 29:40, respectively. Find the monthly income of Aavesh if monthly savings of Axar is 32.5% less than
that of Aavesh.
अक्षर और आवेश की माससक आय क्रमशः 7:10 के अनुपात में है, जबवक उनके माससक व्यय का अनुपात क्रमशः 29:40 है। आवेश की
माससक आय ज्ञात कीसजए, यदद अक्षर की माससक बचत, आवेश की माससक बचत से 32.5% कम है।
A Rs. 32000
B Rs. 27500
C Rs. 24000
D Cannot be determined
E None of these
60. Vivek invested his money in two schemes ‘A’ and ‘B’ in the ratio of 9:5, respectively at same time. Scheme ‘A’
offers simple interest of 25% p.a. while scheme ‘B’ offers compound interest (compounded annually) of 20% p.a. If
the total interest earned by him at the end of 2 years is Rs. 8040, then find the total money invested by him in both
the schemes together.
वववेक ने एक ही समय में क्रमश: 9:5 के अनुपात में दो योजनाओ ां 'A' और 'B' में अपनी रासश को थनवेश वकया। योजना 'A' 25% प्रवत वषस का
साधारण ब्याज प्रदान करती है जबवक योजना 'B' 20% प्रवत वषस चक्रवृद्धि ब्याज (वावषि क देय) प्रदान करती है। यदद 2 वषस के अांत में उसके
द्वारा असजि त कुल ब्याज Rs. 8040 है तो उसके द्वारा दोनों योजनाओ ां में थनवेश की गई कुल धनरासश ज्ञात कीसजए।
A Rs. 12400
B Rs. 16800
C Rs. 15500
D Rs. 17900
E Rs. 15400
61. In a class, there are 50 girls while the number of boys is 10 less than number of girls in it. In the given class,
average weight of a girl is 17 kg more than half of the average weight of a boy. The average weight of the whole
class is 60 kg. Find the average weight of each boy in the class.

22
mathsbyarunsir007@gmail.com WhatsApp @ 8445784192
maths by arun sir maths by arun sir maths by arun sir maths by arun sir
Free Magazines & PDFs for
Quant Updater Banking Aspirants
02 May 2023 Maths by Arun Sir

एक कक्षा में 50 लड़वकयाूँ हैं जबवक लड़कों की सांख्या उसमें लड़वकयों की सांख्या से 10 कम है। दी गई कक्षा में, एक लड़की का औसत वजन
एक लड़के के औसत वजन के आधे से 17 kg अगधक है। पूरी कक्षा का औसत वजन 60 kg है। कक्षा में प्रत्येक लड़के का औसत वजन ज्ञात
कीसजए।
A 60 kg
B 72 kg
C 80 kg
D 64 kg
E 70 kg

31Solution From common explanation, we have


Common explanation : jodhpur= 2000
Let the number of patients in Mumbai , Jodhpur, Calcutta = 800
China ,Calcutta, Gujrat were M, J, Ch, Cal, G
respectively. 33Solution
Then we have From common explanation, we have
M= 1.6J = 1.6(400+Ch) Calcutta = 800
Mumbai = 1500
Total = 2300
G= Ch – 100
Average = 1150
Therefore, we have
34Solution
M + J + Ch + Cal + G = 9100
Common explanation :
For each 1000 tests we have 130 positive.
940 + 5.1Ch = 9100 Thus for 9100 = 70 ( × 130), we should have 70 (×
Ch = 1600 1000) = 70,000 tests.
Thus, patients in various cities are 35Solution
Mumbai = 3200 From common explanation, we have
Jodhpur= 2000 It is given that out of 200 patients, 180 recovered
China = 1600 within 14 days, 18 takes 30 days to recover
Calcutta = 800 Number of patients in Mumbai, Jodhpur and
Gujrat = 1500 Calcutta = 3200, 2000, and 800 = 6000
From common explanation, we have From 30 April to 31 may 180 + 18 = 198 people out of
China = 1600 200 will recovered,
Calcutta = 800
Mumbai = 1500
Total = 3900 Thus, number of people who will recover from the
Average = 1300 three cities = 99% of 6000 = 5940.
32Solution 36Solution
From the common explanation, we have
It is given that out of 200 patients, only 2 dies,

23
mathsbyarunsir007@gmail.com WhatsApp @ 8445784192
maths by arun sir maths by arun sir maths by arun sir maths by arun sir
Free Magazines & PDFs for
Quant Updater Banking Aspirants
02 May 2023 Maths by Arun Sir

Let the number of Mobiles(M) and number of


Chargers (C) sold be ‘9x’ and ‘7x’, respectively
Number of patients in Jodhpur, Gujrat and China =
According to data,
2000 + 1500 + 1600 = 5100
9x – 7x = 190
Number of people who will die = 1% of 5100 = 51
Or, 2x = 190
37Solution
Or, x = 95
For Monday –
Number of Mobiles(M) sold = 9 × 95 = 855
Let the number of Mobiles(M) and number of
Number of Chargers (M) sold = 7 × 95 = 665
Chargers (C) sold be ‘9x’ and ‘7x’, respectively
Total number of items sold = 855 + 665 = 1520
According to data,
Similarly,
9x – 7x = 190
Or, 2x = 190 Number Total
of numbe
Or, x = 95
Number of Charger r of
Number of Mobiles(M) sold = 9 × 95 = 855
Mobiles(M s (C) items
Number of Chargers (M) sold = 7 × 95 = 665 Days ) sold sold sold
Total number of items sold = 855 + 665 = 1520
Similarly, Monday 855 665 1520

Number Total Tuesday 840 600 1440


of numbe
Number of Charger r of
Mobiles(M s (C) items Wednesda
Days ) sold sold sold y 810 450 1260

Monday 855 665 1520 Thursday 780 600 1380

Tuesday 840 600 1440 Friday 880 770 1650

Required average = {(1440 + 1650)/2} = (3090/2) =

Wednesda 1545
y 810 450 1260 Hence, option b.
39Solution
Thursday 780 600 1380 For Monday –
Let the number of Mobiles(M) and number of
Friday 880 770 1650
Chargers (C) sold be ‘9x’ and ‘7x’, respectively
Sum of number of M sold in the Days of Tuesday and
According to data,
Wednesday = 840 + 810 = 1650
9x – 7x = 190
Sum of number C sold in the Days of Wednesday
Or, 2x = 190
and Thursday = 450 + 600 = 1050
Or, x = 95
Required ratio = 1650:1050 = 11:7
Number of Mobiles(M) sold = 9 × 95 = 855
Hence, option d.
Number of Chargers (M) sold = 7 × 95 = 665
38Solution
Total number of items sold = 855 + 665 = 1520
For Monday –

24
mathsbyarunsir007@gmail.com WhatsApp @ 8445784192
maths by arun sir maths by arun sir maths by arun sir maths by arun sir
Free Magazines & PDFs for
Quant Updater Banking Aspirants
02 May 2023 Maths by Arun Sir

Similarly,
Tuesday 840 600 1440
Number Total
of numbe
Number of Charger r of Wednesda
Mobiles(M s (C) items y 810 450 1260
Days ) sold sold sold
Thursday 780 600 1380
Monday 855 665 1520
Friday 880 770 1650
Tuesday 840 600 1440
Total number of C sold in the day of tuesday and
Thursday = 600 + 600 = 1200
Wednesda Required percentage = {(1200 – 840)/1200} × 100 =
y 810 450 1260
30%

Thursday 780 600 1380 Hence, option e.


41Solution
Friday 880 770 1650
For Monday –
Total number of M sold in the day of Thursday and
Let the number of Mobiles(M) and number of
Friday = 780 + 880 = 1660
Chargers (C) sold be ‘9x’ and ‘7x’, respectively
Required difference = 1660 – 1520 = 140
According to data,
Hence, option c.
9x – 7x = 190
40Solution
Or, 2x = 190
For Monday –
Let the number of Mobiles(M) and number of Or, x = 95

Chargers (C) sold be ‘9x’ and ‘7x’, respectively Number of Mobiles(M) sold = 9 × 95 = 855
According to data, Number of Chargers (M) sold = 7 × 95 = 665
9x – 7x = 190 Total number of items sold = 855 + 665 = 1520
Or, 2x = 190 Similarly,
Or, x = 95
Number of Mobiles(M) sold = 9 × 95 = 855 Number Total
of numbe
Number of Chargers (M) sold = 7 × 95 = 665
Number of Charger r of
Total number of items sold = 855 + 665 = 1520
Mobiles(M s (C) items
Similarly, Days ) sold sold sold

Number Total
Monday 855 665 1520
of numbe
Number of Charger r of
Mobiles(M s (C) items Tuesday 840 600 1440
Days ) sold sold sold

Monday 855 665 1520

25
mathsbyarunsir007@gmail.com WhatsApp @ 8445784192
maths by arun sir maths by arun sir maths by arun sir maths by arun sir
Free Magazines & PDFs for
Quant Updater Banking Aspirants
02 May 2023 Maths by Arun Sir

D 1056 864 1920


Wednesda
y 810 450 1260 E 640 640 1280

Required difference = (700 + 1260) – 864 = 1096


Thursday 780 600 1380
Hence, option c.
43Solution
Friday 880 770 1650
For shop A –
Difference between the number of M sold and LeE the total number of writing material sold be ‘8x’
number of C sold in Saturday = 360 – 360 × 0.625 = Number of pencil sold = 8x × 0.625 = 5x
135 According to data given:

Total number of items sold in the day of Saturday = Or, 5x = 700

135 × {(14 + 11)/(14 – 11)} = 1125 Or, x = 140


Therefore, total number of writing material sold = 8x
Hence, option a.
= 1120
42Solution And, number of pen sold = 1120 – 700 = 420
For shop A – Similarly,

LeE the total number of writing material sold be ‘8x’ Total


number of
Number of pencil sold = 8x × 0.625 = 5x
Number Number writing
According to data given:
of pencil of pen material
Or, 5x = 700 Shops sold sold sold

Or, x = 140
A 700 420 1120
Therefore, total number of writing material sold = 8x
= 1120 B 1260 420 1680

And, number of pen sold = 1120 – 700 = 420


C 1560 1440 3000
Similarly,
D 1056 864 1920
Total
number of E 640 640 1280
Number Number writing
Required ratio = (420 + 640):3000 = 1060:3000 =
of pencil of pen material
53:150
Shops sold sold sold
Hence, option b.
44Solution
A 700 420 1120
For shop A –
B 1260 420 1680 LeE the total number of writing material sold be ‘8x’
Number of pencil sold = 8x × 0.625 = 5x
C 1560 1440 3000 According to data given:

26
mathsbyarunsir007@gmail.com WhatsApp @ 8445784192
maths by arun sir maths by arun sir maths by arun sir maths by arun sir
Free Magazines & PDFs for
Quant Updater Banking Aspirants
02 May 2023 Maths by Arun Sir

Or, 5x = 700
material
Or, x = 140
sold
Therefore, total number of writing material sold = 8x
= 1120 A 700 420 1120
And, number of pen sold = 1120 – 700 = 420
Similarly, B 1260 420 1680

Total
C 1560 1440 3000
number of
Number Number writing
D 1056 864 1920
of pencil of pen material
Shops sold sold sold E 640 640 1280

A 700 420 1120 Required average = {(1120 + 1920)/2} = 1520


Hence, option d.
B 1260 420 1680
46Solution

C 1560 1440 3000 For shop A –


LeE the total number of writing material sold be ‘8x’
D 1056 864 1920
Number of pencil sold = 8x × 0.625 = 5x

E 640 640 1280 According to data given:


Or, 5x = 700
Required percentage = {(1056 – 864)/864} × 100 =
Or, x = 140
(200/9)%
Hence, option e. Therefore, total number of writing material sold = 8x

45Solution = 1120

For shop A – And, number of pen sold = 1120 – 700 = 420


LeE the total number of writing material sold be ‘8x’ Similarly,
Number of pencil sold = 8x × 0.625 = 5x
Total
According to data given:
number of
Or, 5x = 700
Number Number writing
Or, x = 140 of pencil of pen material
Therefore, total number of writing material sold = 8x Shops sold sold sold
= 1120
And, number of pen sold = 1120 – 700 = 420 A 700 420 1120

Similarly,
B 1260 420 1680
Number Number Total
C 1560 1440 3000
of pencil of pen number of
Shops sold sold writing D 1056 864 1920

27
mathsbyarunsir007@gmail.com WhatsApp @ 8445784192
maths by arun sir maths by arun sir maths by arun sir maths by arun sir
Free Magazines & PDFs for
Quant Updater Banking Aspirants
02 May 2023 Maths by Arun Sir

Sanjeev’s monthly income = Rs 64000/2 = Rs 32000


E 640 640 1280
Let the Rahul’s monthly income be Rs x
Number of pen sold by shop G = 1.25 × 1440 = 1800 Now, according to the question,
Therefore, required difference = 1800 – 420 = 1380 4/7*x = 32000/2
Hence, option e. or, 4x = 16000 × 7
47SOL. or, x = (16000*7)/4 = 28000
Part of tank emptied in one minute when all the Rahul’s monthly income = Rs 28000
pipes are opened = 1/27 + 1/18 - 1/6 52Solution
= 2/27 Let the initial investment made by Vinit and Punit be
So 2/3 of tank will be emptied in 27/2 * 2/3 minutes Rs. 7x and Rs. 9x, respectively
= 9 minutes Profit sharing ratio of Vinit and Punit = {7x × 8 + (7x –
48SOL. 500) × 4}:{9x × 8 + (9x + 500) × 4}
Total marks obtained by Radhika 1400:3360 = (84x – 2000):(108x + 2000)
= 43 + 45 + 67 + 89 + 65 = 309 Or, 5:7 = (21x – 500):(27x + 500)
309
∴ Percentage of marks obtained = × 100 = 51.5 Or, 135x + 2500 = 147x – 3500
600
49Sol. Or, 12x = 6000
Sagar can row 12 km in 7 hrs. in downstream and 8 Or, x = 500
km in 5 hrs. Initial investment of Punit = 9 × 500 = Rs. 4500
So, Sagar’s speed in downstream = 12/7 km/hr. Hence, option b.
And, Sagar’s speed in upstream = 8/5 km/hr. 53Solution
We know that if the speed of the downstream is x Let quantity of milk and water which were poured
km/hr and the speed of the upstream is y km/hr, from jar ‘A’ into jar ‘B’ be 17x litres and 11x litres,
then the speed in still water = ½ × (x + y) km/hr. respectively
So, Sagar’s speed in still water = ½ × [(12/7) + (8/5)] According to question:
km/hr. {(17x + 21)/(11x + 75)} = 2/3
= ½ × (116/35) km/hr. Or, 51x + 63 = 22x + 150
= (58/35) km/hr. Or, 29x = 87
∴ The time required to cover 116 km in still water by Or, x = 3
him = 116/(58/35) = 70 hrs. Therefore, initial quantity of mixture in jar ‘A’ = {(17x +
50Sol. 11x)/0.6} = (84/0.6) = 140 litres
Let, age of Sandeep = ‘x’ years Hence, option d.
Age of Mukesh = ‘x+6’ years 54Solution
Age of Ramesh = ‘x+14’ years Let total work = 224 units (LCM of 32 and 28)
So, Efficiency of ‘P’ = 224/32 = 7 units/day
𝑥 5
𝑥+14
=
12
Efficiency of ‘Q’ = 224/28 = 8 units/day
12𝑥 = 5𝑥 + 70 Work done by ‘P’ in 2 days = 2 × 7 = 14 units
7𝑥 = 70, 𝑥 = 10
Work done by ‘Q’ on 3rd day = 8 units
Age of Mukesh after 12 years = 𝑥 + 6 + 12 = 28 years
Total work done in 3 days = 14 + 8 = 22 units
51Sol.

28
mathsbyarunsir007@gmail.com WhatsApp @ 8445784192
maths by arun sir maths by arun sir maths by arun sir maths by arun sir
Free Magazines & PDFs for
Quant Updater Banking Aspirants
02 May 2023 Maths by Arun Sir

Total work done in 30 days i.e. (3 × 10) days = 22 × 10 Or, 2P = 960


= 220 units Or, P = 480
Amount of work left = 224 – 220 = 4 units Selling price of the article = 2400 + 480 = Rs. 2880
Time taken by ‘P’ to complete 4 units of work = (4/7) Hence, option d.
days 57Solution
Total time taken to complete the work this way = 30 Let the radius and slant height of the cone be ‘r’ cm
+ (4/7) = (214/7) days and ‘l’ cm, respectively
Hence, option a. According to question:
55Solution πrl = 408……. (I)
2 years hence from now, let the age of Mina and Rina And, πrl + πr2 = 600…… (II)
be ‘8x’ years and ‘9x’ years, respectively. Subtracting equation (I) from equation (II), we get
Present age of Mina = (8x – 2) years Or, πr2 = 600 – 408
Present age of Rina = (9x – 2) years Or, r2 = 192/3
Sum of present age Tina and Rina = (30 × 2) = 60 Or, r2 = 64
years Or, r = 8 (Since, radius cannot be negative therefore
Present age of Tina = 60 – (9x – 2) = (62 – 9x) years we will take positive root only)
According to question: Height of the cylinder = 15 cm
{(62 – 9x + 2)/(8x – 2 – 9)} = 4/3 Radius of cylinder = 8 + 8 × 0.75 = 14 cm
Or, 186 – 27x + 6 = 32x – 8 – 36 Volume of cylinder = π × radius2 × height = 3 × 14 × 14
Or, 192 – 27x = 32 – 44 × 15 = 8820 cm3
Or, 59x = 236 Hence, option e.
Or, x = 4 58Solution
Present age of Rina = 9x – 2 = (9 × 4) – 2 = 34 years Speed of train B = 54 × (5/18) = 15 metre/sec
Hence, option b. Let the length of train ‘A’ and train ‘B’ be ‘l1’ metre and
56Solution ‘l2’ metre
Let the discount offered and profit earned be ‘Rs. D’ And speed of train ‘A’ be ‘s1’ metre/sec
and Rs. P According to question:
Marked price of the article = 2400 + 2400 × 0.5 = (l1 + l2)/(s1 + 15) = 8 ….. (eq.1)
Rs.3600 And, (l1 + l2)/(s1 – 15) = 32 ….. (eq.2)
According to question: After solving (eq.1) and (eq.2)
D = P + 240 …. (I) (s1 – 15)/(s1 + 15) = 8/32
And, D = Marked price – Selling price 4s1 – 60 = s1 + 15
P = Selling price – Cost price 3s1 = 75
So, D + P = Marked price – Selling price + Selling price s1 = 25 metre/sec = 25 × (18/5) = 90 km/hr
– Cost price Required distance travelled = 90 × 5 = 450 km
Or, D + P = Marked price – Cost price Hence, option c.
Or, D + P = 3600 – 2400 = 1200 …. (II) 59Solution
After putting the value of ‘P’ from equation (I), we get Let the monthly incomes of Axar and Aavesh be ‘Rs.
Or, P + P + 240 = 1200 7x’ and ‘Rs. 10x’, respectively

29
mathsbyarunsir007@gmail.com WhatsApp @ 8445784192
maths by arun sir maths by arun sir maths by arun sir maths by arun sir
Free Magazines & PDFs for
Quant Updater Banking Aspirants
02 May 2023 Maths by Arun Sir

And, monthly expenditure of Axar and Aavesh be Rs. (9x/2) + (11x/5) = 8040
‘29y’ and Rs. ‘40y’, respectively Or, (67x/10) = 8040
Ratio of savings of Axar and Aavesh = {(100 – Or, x = (8040/67) × 10
32.5)/100} = (67.5/100) = (27/40) Or, x = 1200
According to question: Total money invested by Vivek in both the schemes
{(7x – 29y)/(10x – 40y)} = 27/40 = 9x + 5x = (9 + 5) × 1200 = Rs. 16800
Or, 280x – 1160y = 270x – 1080y Hence, option b.
Or, x = 8y 61Solution
So, we cannot find the monthly income of Aavesh. Number of boys in the school = 50 – 10 = 40
Hence, option d. Let the average weight of a boy be ‘2x’ years
60Solution So, average weight of a girl = (x – 17) years
Let the money invested by Vivek in scheme ‘A’ and According to question:
scheme ‘B’ be Rs. 9x and Rs. 5x, respectively 40 × (2x) + 50 × (x + 17) = 60 × (50 + 40)
Interest earned from scheme ‘A’ = (9x × 25 × 2)/100 = Or, 130x = 4550
Rs. (9x/2) Or, x = 35
Compound interest earned from scheme ‘B’ = 5x × {(1 Average weight of a boy = 2 × 35 = 70 kg
+ 20/100)2 – 1} = Rs. (11x/5) Hence, option e.
According to question:

30
mathsbyarunsir007@gmail.com WhatsApp @ 8445784192
maths by arun sir maths by arun sir maths by arun sir maths by arun sir
Free Magazines & PDFs for
Quant Updater Banking Aspirants
02 May 2023 Maths by Arun Sir

31
mathsbyarunsir007@gmail.com WhatsApp @ 8445784192
maths by arun sir maths by arun sir maths by arun sir maths by arun sir
Free Magazines & PDFs for
Quant Updater Banking Aspirants
02 May 2023 Maths by Arun Sir

32
mathsbyarunsir007@gmail.com WhatsApp @ 8445784192
maths by arun sir maths by arun sir maths by arun sir maths by arun sir

You might also like